feat: aggiunge le basi degli appunti del 03/04 e del 04/04/2023

main
parent 74b3f47e97
commit 4e04028baf

@ -165,4 +165,129 @@
Dimostrare che la derivata sinistra è negativa, e che quella
destra è positiva nei casi che hai capito.
\end{example}
\end{document}
\begin{theorem} (di Rolle)
Sia $I = [a, b] \subset \RR$ e sia $f : I \to \RR$ tale che
$f$ sia continua su $I$, che $f(a) = f(b)$ e che $f$ sia derivabile
in $[a, b]$. Allora $\exists \xbar \in (a, b)$ tale che $f'(\xbar) = 0$.
\end{theorem}
\begin{proof}
Per il teorema di Weierstrass $f$ ammette un punto di massimo $M$ e uno di minimo $m$ in $I$. Se $f(a) = M$ e $f(b) = m$ o viceversa, la
funzione $f$ è costante in $I$, e quindi per ogni punto in $(a, b)$
la derivata è nulla, dacché $f$ è sempre derivabile. Altrimenti,
sicuramente uno tra il punto di massimo e quello di minimo appartiene
a $(a, b)$. Senza perdita di generalità, si assuma che $\exists x_M \in (a, b)$ tale che $f(x_M) = M$: per
il teorema di Fermat $f'(x_M) = 0$. Analogamente per il caso in cui
$\exists x_m \in (a, b)$ tale che $f(x_m) = m$, da cui la tesi.
\end{proof}
\begin{theorem} (di Cauchy)
Sia $I = [a, b] \subset \RR$ e siano $f$, $g: I \to \RR$
continue su $I$ e derivabili in $(a, b)$, con $g'$ non nulla
in $(a, b)$ e $g(a) \neq g(b)$. Allora
$\exists \xbar \in (a, b)$ tale che $\frac{f'(\xbar)}{g'(\xbar)} = \frac{f(b) - f(a)}{g(b)-g(a)}$.
\end{theorem}
\begin{proof}
Si consideri la funzione $h : I \to \RR$ tale che $h(x) = f(x) - \left(\frac{f(b) - f(a)}{g(b) - g(a)} (g(x) - g(a)) + f(a)\right)$.
Si osserva che $h$,
essendo una somma di funzioni continue su $I$ e derivabili in $(a, b)$,
è anch'essa continua su $I$ e derivabile in $(a, b)$. Inoltre
$h(a) = h(b) = 0$. Quindi, per il teorema di Rolle, $\exists \xbar \in (a, b) \mid h'(\xbar) = 0 \implies \frac{f'(\xbar)}{g'(\xbar)} = \frac{f(b)-f(a)}{g(b)-g(a)}$,
da cui la tesi.
\end{proof}
\begin{theorem} (di Lagrange)
Sia $I = [a, b] \subset \RR$ e sia $f: I \to \RR$ tale che $f$
sia continua su $I$ e che $f$ sia derivabile in $(a, b)$. Allora
$\exists \xbar \in (a, b)$ tale che $f'(\xbar) = \frac{f(b) - f(a)}{b-a}$, ossia la cui retta tangente è parallela alla secante
che passa per $(a, f(a))$ e $(b, f(b))$.
\end{theorem}
\begin{proof}
Si consideri $g(x) = x$, $g$ è continua in $[a, b]$ e derivabile
in $(a, b)$, con derivata sempre non nulla in tale intervallo.
Allora, per il teorema di Cauchy, $\exists \xbar \in (a, b) \mid
f'(\xbar) = \frac{f(b)-f(a)}{b-a}$, da cui la tesi.
\end{proof}
\begin{proposition}
Sia $I = [a, b] \subset \RR$ e sia $f : I \to \RR$ tale che $f$
sia continua su $I$ e che $f$ sia derivabile in $(a, b)$, con
derivata non negativa. Allora $f$ è crescente in $[a, b]$.
Analogamente, se la derivata è non positiva, $f$ è decrescente.
\end{proposition}
\begin{proof}
Senza perdita di generalità si dimostra il caso in cui la derivata
di $f$ in $(a, b)$ è non negativa (altrimenti è sufficiente considerare
$g = -f$).
Si considerino $c < d \in I$. Allora, per il teorema di Lagrange,
$\exists \xbar \in (c, d) \mid f'(c) = \frac{f(d) - f(c)}{d-c}
\implies f(d) - f(c) = \underbrace{f'(c) (d-c)}_{\geq 0} \implies
f(d) \geq f(c)$, ossia che $f$ è crescente in $I$.
\end{proof}
\begin{remark}\nl
\li L'interpretazione geometrica del teorema di Cauchy, rispetto
a quella di Lagrange, è leggermente più complicata. Si consideri
la curva $\gamma : \RR \to \RR^2$ tale che
$\gamma(t) =(g(t), f(t))$. Si osserva che il coefficiente della
retta tangente in $\xbar$ per $\gamma$ è dato da $\lim_{h \to 0} \frac{f(\xbar + h) - f(\xbar)}{g(\xbar + h) - g(\xbar)}$, che,
sotto le ipotesi del teorema di Cauchy, può essere riscritto
come $\frac{f'(\xbar)}{g'(\xbar)}$. Allora, il teorema di Cauchy
asserisce che esiste un punto della curva $\gamma$ tale per cui
la retta tangente alla curva in quel punto è parallela alla secante
passante per $(g(a), f(a))$ e $(g(b), f(b))$.
\end{remark}
\begin{exercise}
Dare un esempio di una funzione $f : \RR \to \RR$ crescente e
discontinua $\forall x \in \ZZ$.
\end{exercise}
\begin{solution}
Si consideri $f(x) = \lfloor x \rfloor$.
\end{solution}
\begin{exercise}
Si descriva un insieme $X$ tale che i suoi punti di accumulazione
sono $\{\pm 1\}$.
\end{exercise}
\begin{solution}
Si consideri $X = \{1 + \frac{1}{n}\} \cup \{-1 + \frac{1}{n}\}$.
\end{solution}
\begin{exercise}
Sia $f : X \to \RRbar$ continua in $\xbar$ e sia $a < f(\xbar)$.
Allora esiste $J$ intorno di $\xbar$ tale che $a < f(x)$ $\forall
x \in J$.
\end{exercise}
\begin{exercise}
Sia $X \subseteq \RRbar$ e sia $\xbar$ punto di accumulazione di $X$,
$f_1$, $f_2 : X \to \RRbar$. Allora:
\begin{enumerate}[(i)]
\item Se $f_1 \tendsto{\xbar} +\infty$ e $f_2$ è limitata inferiormente
in un intorno $J$ di $\xbar$, allora $f_1(x) + f_2(x) \tendsto{\xbar} +\infty$.
\item Se $f_1 \tendsto{\xbar} 0$ e $f_2$ è limitata in un intorno
di $\xbar$, allora $f_1 f_2(x) \tendsto{\xbar} 0$.
\item Se $f_1 \tendsto{\xbar} +\infty$ è limitata inferiormente
da una costante positiva $m$ in un intorno $J$ di $\xbar$, allora
$f_1 f_2 \tendsto{\xbar} +\infty$.
\end{enumerate}
\end{exercise}
\begin{exercise}
Sia $f: \RR \to \RR$ tale che:
\[ f(x) = \system{x + 2x^2 \sin\left(\frac{1}{x}\right) & \se x \neq 0, \\ 0 & \altrimenti.} \]
\vskip 0.05in
Mostrare che $f$ è continua, che $f'(0) = 1$ e che $f'$ non è continua in zero.
\end{exercise}
\end{document}

@ -0,0 +1,37 @@
\documentclass[11pt]{article}
\usepackage{personal_commands}
\usepackage[italian]{babel}
\title{\textbf{Note del corso di Geometria 1}}
\author{Gabriel Antonio Videtta}
\date{\today}
\begin{document}
\maketitle
\begin{center}
\Large \textbf{Esercitazione: forma canonica di Jordan reale}
\end{center}
\begin{exercise}
Sia $M \in M(n, \RR)$ tale che $\exists a_1$, ..., $a_k \in \RR$ distinti
tale che:
\[ (M^2 + a_1^2 I) \cdots (M^2 + a_k^2 I) = 0. \]
Dimostrare allora che esistono $S$, $A \in M(n, \RR)$ tale che
$M = SA$ con $S$ simmetrica e $A$ antisimmetrica.
\end{exercise}
\begin{solution}
Per ipotesi, $p(x) = (x^2+a_1^2) \cdots (x^2 + a_k^2) \in \Ker \sigma_M$.
Dal momento che $p(x)$ si scompone in fattori lineari distinti in
$\CC$, $p(x)$ è anche il polinomio minimo di $M$. Si deduce
allora che $M$ è diagonalizzabile, e che i suoi autovalori sono
esattamente $\pm a_1 i$, ..., $\pm a_k i$. Allora la forma
canonica di Jordan reale di $M$ è:
\[ J = \Matrix{} \]
\end{solution}
\end{document}
Loading…
Cancel
Save